- PowerScore Staff
- Posts: 5972
- Joined: Mar 25, 2011
- Fri Nov 03, 2017 2:17 pm
#41158
Complete Question Explanation
(The complete setup for this game can be found here: lsat/viewtopic.php?t=11758)
The correct answer choice is (E)
This is the type of question where one might expect that J and K would be the correct answer (if J and K were in the same class as F, there would not be a third student for the level 1 class). However, that answer does not appear among the five lettered choices. Instead, we will need to find a different pair that creates a problem.
Answer choice (A) is incorrect because F, H, and N can be in the level 3 class, leaving L, M, and J or K for the level 2 class, and I, G and K or J for the level 1 class. Thus, as this student pair produces a workable solution, this answer choice is incorrect.
Answer choice (B) is incorrect because J, L, and F can be in the level 2 class, leaving M, H and N in the level 3 class, and I, G and K for the level 1 class. Thus, as this student pair produces a workable solution, this answer choice is incorrect.
Answer choice (C) is incorrect because K, L, and F can be in the level 3 class, leaving M, H and N for the level 2 class, and I, G and J for the level 1 class. Thus, as this student pair produces a workable solution, this answer choice is incorrect.
Answer choice (D) is incorrect because J, M, and F can be in the level 2 class, leaving L, H and N in the level 3 class, and I, G and K for the level 1 class. Thus, as this student pair produces a workable solution, this answer choice is incorrect.
In answer choice (E), placing F with L and M leaves only H and N for the level 3 class. Since each level must have three students, the scenario in answer choice (E) cannot occur, and therefore (E) is correct.
(The complete setup for this game can be found here: lsat/viewtopic.php?t=11758)
The correct answer choice is (E)
This is the type of question where one might expect that J and K would be the correct answer (if J and K were in the same class as F, there would not be a third student for the level 1 class). However, that answer does not appear among the five lettered choices. Instead, we will need to find a different pair that creates a problem.
Answer choice (A) is incorrect because F, H, and N can be in the level 3 class, leaving L, M, and J or K for the level 2 class, and I, G and K or J for the level 1 class. Thus, as this student pair produces a workable solution, this answer choice is incorrect.
Answer choice (B) is incorrect because J, L, and F can be in the level 2 class, leaving M, H and N in the level 3 class, and I, G and K for the level 1 class. Thus, as this student pair produces a workable solution, this answer choice is incorrect.
Answer choice (C) is incorrect because K, L, and F can be in the level 3 class, leaving M, H and N for the level 2 class, and I, G and J for the level 1 class. Thus, as this student pair produces a workable solution, this answer choice is incorrect.
Answer choice (D) is incorrect because J, M, and F can be in the level 2 class, leaving L, H and N in the level 3 class, and I, G and K for the level 1 class. Thus, as this student pair produces a workable solution, this answer choice is incorrect.
In answer choice (E), placing F with L and M leaves only H and N for the level 3 class. Since each level must have three students, the scenario in answer choice (E) cannot occur, and therefore (E) is correct.
Dave Killoran
PowerScore Test Preparation
Follow me on X/Twitter at http://twitter.com/DaveKilloran
My LSAT Articles: http://blog.powerscore.com/lsat/author/dave-killoran
PowerScore Podcast: http://www.powerscore.com/lsat/podcast/
PowerScore Test Preparation
Follow me on X/Twitter at http://twitter.com/DaveKilloran
My LSAT Articles: http://blog.powerscore.com/lsat/author/dave-killoran
PowerScore Podcast: http://www.powerscore.com/lsat/podcast/